Difference between revisions of "1979 USAMO Problems/Problem 1"
Viperstrike (talk | contribs) (→Solution 2) |
Viperstrike (talk | contribs) (→Solution 2) |
||
Line 9: | Line 9: | ||
== Solution 2 == | == Solution 2 == | ||
− | By AM-GM, <math>\dfrac{n^{4}_1+...+n^{4}_{14}}{14}\geq(n_1...n_{14})^{2/7}</math>. Assume there exist <math>n_i</math> which satisfy the given equation. Then <math>(n_1...n_{14})\leq | + | By AM-GM, <math>\dfrac{n^{4}_1+...+n^{4}_{14}}{14}\geq(n_1...n_{14})^{2/7}</math>. Assume there exist positive integers <math>n_i</math> which satisfy the given equation. Then <math>(n_1...n_{14})\leq{\dfrac{1599}{14}}^{7/2}<2</math>. So <math>n_1...n_{14}=1</math> and <math>n_1=...=n_{14}=1</math>, clearly a contradiction. Thus there are no solutions to the given equation. |
== See also == | == See also == |
Revision as of 13:30, 12 August 2012
Contents
Problem
Determine all non-negative integral solutions if any, apart from permutations, of the Diophantine Equation .
Solution 1
Alternate solutions are always welcome. If you have a different, elegant solution to this problem, please add it to this page.
Recall that for all integers . Thus the sum we have is anything from 0 to 14 modulo 16. But , and thus there are no integral solutions to the given Diophantine equation.
Solution 2
By AM-GM, . Assume there exist positive integers which satisfy the given equation. Then . So and , clearly a contradiction. Thus there are no solutions to the given equation.
See also
1979 USAMO (Problems • Resources) | ||
Preceded by First Question |
Followed by Problem 2 | |
1 • 2 • 3 • 4 • 5 | ||
All USAMO Problems and Solutions |